Uncertainty error of resistors in parallel

In summary: The worst possible combination is the one that happens if the two components vary in such a way that their total resistance is maximum. This is the case if R1 is at its upper limit and R2 is at its lower limit, or R1 is at its lower limit and R2 is at its upper limit. In both cases, the total resistance is 1116 ohms. The teacher's method would produce a range of 853-1116 ohms, which is too large.## \delta_{r_x}## is not equal to ##\delta_x / x## because it's not a relative error. It's just the error in ##r_x##. Similarly, ## \delta_{x+y}## is not equal
  • #1
valiantt
1
0

Homework Statement



Find the Uncertainty of the Total Resistance of two (2) parallel-connected resistors:
[tex]R1 = 2.2 kΩ ± 5\% \ ;\ R2 = 1.8 kΩ ± 20\%[/tex].

Homework Equations



We know that resistors in parallel are:

[tex]\frac{1}{R_t}={\frac{1}{R_1±\delta_{R_1}}+\frac{1}{R_2±\delta_{R_2}}}[/tex]

This implies: (Eq. 1)
[tex]R_t=\frac{1}{\frac{1}{R_1±\delta_{R_1}}+\frac{1}{R_2±\delta_{R_2}}}[/tex]

Which is also equal to: (Eq. 2)
[tex]R_t=\frac{(R_1±\delta_{R_1})(R_2±\delta _{R_2})}{( R_1±\delta_{R_1}) + (R_2±\delta _{R_2)}}[/tex]

The Attempt at a Solution



According to rules established by our teacher:

In addtion: if given [tex] f±\delta_f = (x±\delta_x)+(y±\delta_y)[/tex].
it implies:
[tex]f±\delta_f = (x+y)±(\delta_x+\delta_y)[/tex]
This means that in addition, δf is equal to the sum of the absolute errors.

In multiplication: if given [tex] f±\delta_f = (x±\delta_x)(y±\delta_y)[/tex].
it implies:
[tex]f±\frac{\delta_f}{f} = (x*y)±(\frac{\delta_x}{x}+\frac{\delta_y}{y})[/tex]

From here on, let's established for simplification that:

[tex]\frac{\delta_f}{f} = \%Err_x[/tex]

This means that in multiplication, δf is equal to the sum of the relative errors. (Note that this also applies to division)-----------------------------------------------------------------------

So given those above, when should those rules be applied? My teacher insists that you should apply those rules at Eq. 2. This means his method yields:

[tex] R_t = \frac{(R_1*R_2) ± (\%Err_{R_1}+\%Err_{R_2})}{(R_1+R_2) ± (R_1 * \%Err_{R_1}+R_2*\%Err_{R_2})} [/tex]

[tex]\Rightarrow \frac{2.2kΩ * 1.8kΩ ± (5\% + 20\%)} {2.2kΩ + 1.8kΩ ± (110Ω + 360Ω)} = \frac{3.96MΩ±25\%}{4000 ±(470Ω)} = \frac{3.96MΩ±25\%}{4000Ω ±11.75\%}[/tex]

[tex]\Rightarrow \frac{3.96MΩ}{4kΩ }±(25\%+11.75\%) 990Ω±36.75\%[/tex]

[tex]\Rightarrow990Ω±36.75\% [/tex]But according to this link, http://uregina.ca/~szymanss/uglabs/companion/Ch3_Error_Prop.pdf , Eq. 2 is not in a appropriate form for error calculation. It then shows that:

[tex]if \quad r_x = \frac{1}{x} ,~ then\quad \frac{\delta_{r_x}}{r_x} = \frac{\delta_{x}}{x}[/tex]

In this case, it says that the relative error is unchanged if you take the reciprocal of a quantity. This is also supported by this link: http://ipl.physics.harvard.edu/wp-uploads/2013/03/PS3_Error_Propagation_sp13.pdf

Now using Eq. 1 and the reciprocality rule:

[tex]R_t=\frac{1}{\frac{1}{R_1±\delta_{R_1}}+\frac{1}{R_2±\delta_{R_2}}} = \frac{1}{\frac{1}{R_1}±\%Err_{R_1}+\frac{1}{R_2}±\%Err_{R_2}}[/tex]

[tex]\Rightarrow \frac{1}{\frac{1}{2.2kΩ±5\%}+\frac{1}{1.8kΩ±20\%}} = \frac{1}{\frac{1}{2.2kΩ}±5\%+\frac{1}{1.8kΩ}±20\%}[/tex]

[tex]\Rightarrow \frac{1}{\frac{1}{2.2kΩ}+\frac{1}{1.8kΩ} ± (\frac{1}{44kΩ}+\frac{1}{9kΩ})} = \frac{1}{\frac{1}{990}±\frac{53}{39600}} = \frac{1}{\frac{1}{990}±13.25\%}[/tex]

[tex]\Rightarrow990Ω±13.25\% [/tex]That's pretty much it. But I do remember someone telling me that uncertainty error should always increase so I don't know...

And also, if assuming that the second case is correct, would that mean given a bunch of resistors, say for example, 1 pc. of a 1kΩ ±20% and 99 pcs. of 1kΩ ±5% resistors, all connected in parallel, what would happen? I tried making an equation but I'm not sure if I got the math right...

[tex] R_t = \frac{1}{\frac{1}{1kΩ}*100} ± \frac{\frac{1}{1kΩ}*(0.20+(0.05*99)}{\frac{1}{1kΩ}*100} = 10Ω±5.15\%[/tex]

lastly, given enough 1kΩ± 5% resistors to parallel, would that mean that the %error value will approach 5%?
Not really part of the homework but just a thought experiment. =D
 
Last edited by a moderator:
Physics news on Phys.org
  • #2
Your teacher is wrong. To demonstrate this, try calculating ##R_t## using the extreme values for ##R_1## and ##R_2##. I found a range for ##R_t## from 853 ohms to 1116 ohms, which is consistent with the 13% error you obtained using the second method. Your teacher's method grossly overestimates the error.
 
  • #3
Both calculations are corrected. The worst possible combination is the key to report the worst possible range to others so that your teacher has no mistake. However, when you stand on manufacturer side, you may report the lowest one.
 
  • #4
valiantt said:
##
\Rightarrow \frac{1}{\frac{1}{2.2kΩ}+\frac{1}{1.8kΩ} ± (\frac{1}{44kΩ}+\frac{1}{9kΩ})} = \frac{1}{\frac{1}{990}±\frac{53}{39600}} = \frac{1}{\frac{1}{990}±13.25\%}##
Your notation is rather confusing. Sometimes your r±x means r(1±x) and at other times literally r±x.
This item is particularly puzzling: ##\frac{1}{\frac{1}{990}±\frac{53}{39600}}##.
I think you mean ##\frac{1}{\frac{1}{990}±\frac{53\%}{39600}}##.
However, 13.25% is so large that you can't really use ## \frac{\delta_{r_x}}{r_x} = \frac{\delta_{x}}{x}##. If you don't use that you get a touch over 15%.

As @vela notes, your teacher's method is unnecessarily pessimistic. It overlooks that the errors that make the numerator largest also make the denominator largest, so they partly cancel.
 
  • #5
MC Wong said:
Both calculations are corrected. The worst possible combination is the key to report the worst possible range to others so that your teacher has no mistake. However, when you stand on manufacturer side, you may report the lowest one.
No, the teacher's calculation is wrong, as demonstrated.
 

1. What is the uncertainty error of resistors in parallel?

The uncertainty error of resistors in parallel refers to the potential variance in the measured resistance value when multiple resistors are connected in parallel in an electrical circuit.

2. How is the uncertainty error of resistors in parallel calculated?

The uncertainty error of resistors in parallel can be calculated by using the formula: 1/Req = 1/R1 + 1/R2 + ... + 1/Rn, where Req is the equivalent resistance and R1, R2, etc. are the individual resistances.

3. What factors contribute to the uncertainty error of resistors in parallel?

The uncertainty error of resistors in parallel is influenced by several factors, including the accuracy of the resistor values, the precision of the measuring instrument, and the resistance tolerances of the individual resistors.

4. How can the uncertainty error of resistors in parallel be minimized?

The uncertainty error of resistors in parallel can be reduced by using resistors with tighter tolerance values, using more precise measuring instruments, and taking multiple measurements to calculate an average resistance value.

5. What are the practical applications of understanding the uncertainty error of resistors in parallel?

Understanding the uncertainty error of resistors in parallel is important in various fields, such as electrical engineering, where accurate resistance measurements are crucial for designing and troubleshooting circuits. It also helps in ensuring the reliability of data collected in scientific experiments and research.

Similar threads

  • Introductory Physics Homework Help
Replies
7
Views
751
  • Introductory Physics Homework Help
Replies
4
Views
776
  • Introductory Physics Homework Help
Replies
1
Views
1K
  • Introductory Physics Homework Help
Replies
4
Views
14K
  • Introductory Physics Homework Help
Replies
4
Views
2K
  • Introductory Physics Homework Help
Replies
2
Views
1K
  • Introductory Physics Homework Help
Replies
5
Views
914
  • Introductory Physics Homework Help
Replies
5
Views
2K
  • Introductory Physics Homework Help
Replies
3
Views
5K
  • Electromagnetism
Replies
5
Views
2K
Back
Top